Download as pdf or txt
Download as pdf or txt
You are on page 1of 70

www.byjusexamprep.

com

Mock Test Solutions in English

Questions

1. Direction: In Asimov’s science fiction writing, he is perhaps most well-known for his Three Laws of
Robotics: “1) A robot may not injure a human being or, through inaction, allow a human being to come to
harm. 2) A robot must obey the orders given to it by human beings except where such orders would
conflict with the First Law. 3) A robot must protect its own existence as long as such protection does not
conflict with the First or Second Laws.”

I, Robot, a short story collection published in 1950, is one of Asimov’s most famous works. He wrote
several novels and short stories exploring the interaction between humans and robots, stories that should
receive renewed interest with the advent of artificial intelligence. His short story “Robot Dreams” may be
the scariest science fiction story ever written. Asimov’s Foundation series of 7 novels introduced the
concept of psychohistory, an attempt to predict the future using a combination of sociology and
mathematics.

Isaac Asimov’s legacy includes many books that readers enjoy for both their style and insights. If Isaac
Asimov’s family had not immigrated to America, Asimov speculated he would have died as a Soviet soldier
during World War II. In the days of socialist realism, where literature was meant to serve state and party
interests, it is almost certain Soviet authorities would never have allowed Isaac Asimov to publish science
fiction stories, or at least any of the ones he published in America.

In his three-book “Galactic Empire” series, his characters argue strongly for self-determination and against
allowing a bigger entity to dictate how people in a smaller land should live their lives. Given Soviet
domination of Eastern Europe after World War II, stories employing those themes would have earned
Asimov, at minimum, a stay in Siberia.

“He was very much an American and appreciated our country’s freedoms,” said Sheila Williams. “He was
very sympathetic to the underdog, to small business owners, and thought America was a great place
because it allowed people of all backgrounds to contribute to the nation.”

In works such as The End of Eternity and The Stars, Like Dust, Asimov came down firmly on the side of
human liberty and the freedoms embodied in the US Constitution and the Bill of Rights. Isaac Asimov
www.byjusexamprep.com

immigrated from a place that did not cherish human liberty and freedom, which encouraged him, like
millions of immigrants before and since, to embrace those values even more.

Which of the following most accurately represents the main point of the passage?
A. Isaac Asimov was a family immigrant to America
who believed in human liberty and whose works of B. Isaac Asimov believed in self-determination and
science fiction are relevant in our age of artificial liberty only because of his immigrant status.
intelligence.
C. Asimov escaped to America as his works of
D. Asimov could become a writer only because he
science fiction resulted in an order for his
immigrated to America.
imprisonment.
2. Which of the following most accurately represents the purpose of the final paragraph of the passage?
A. To predict Asimov’s future if he had not emigrated B. To describe the impact of the Asimov’s science
to America fiction
C. To describe the values that Asimov cherished and D. To illustrate the renewed importance of Asimov’s
the reasons for that works in the age of artificial intelligence
3. The author’s reference to the Galactic Empire serves to:
B. Demonstrate his support for the idea that a bigger
A. Support his claim that Asimov had a totalitarian
entity should dictate how people in a smaller land
streak in him
should live their lives
C. Support his claim about the individualistic values
that Asimov cherished and displayed through his D. Support his claim about the Soviet authorities
characters
4. Which of the following best describes the word ‘psychohistory’ in the context of the passage?
A. Predicting the history of psychology using B. Predicting the future using mathematics and
mathematics sociology
C. Predicting the future of science fiction using
D. The science of prediction using psychology
psychology and sociology
5. What can be inferred from the passage about Asimov’s Three Laws of Robotics?
A. Asimov’s robots served and protected human
B. Asimov’s robots were superior to human beings.
beings.
C. The laws were introduced to protect human D. The laws were a psychological assurance against
www.byjusexamprep.com

beings from robots who were trying to gain control. the might of the robots.
6. Direction: In each of the following questions, a word is highlighted. Choose the word that is a synonym of
the highlighted word.

The loquacious taxi driver told me his entire life history during the drive to the airport.
A. irritating B. curious
C. talkative D. foolish
7. Climate change is an ineluctable fact to deal with, and global leadership is in denial of this fact.
A. unpleasant B. inevitable
C. unequivocal D. ineffable
8. Despite being a democracy, India has a tradition of political parties with arrogant leaders and obsequious
supporters blindly following the orders of the former.
A. servile B. haughty
C. supercilious D. resolute
9. Investigations revealed that the new doctor in the area was a charlatan with a fake degree who was
wanted in a case of medical malpractice in the neighbouring state.
A. charmer B. nurse
C. quack D. convict
10. The industrialist is known for his luxurious lifestyle and personal ostentation.
A. mildness B. simplicity
C. modesty D. flamboyance
11. Lata Mangeshkar is India’s most venerated singer.
A. prolific B. respected
C. popular D. prodigious
12. It was heart-wrenching to see the pictures of the drought-ridden area with emaciated children and
starving old people.
A. obese B. corpulent
C. rotund D. scrawny
13. She impressed everyone with her impeccable style.
A. impecunious B. faultless
C. garish D. meretricious
www.byjusexamprep.com

14. I was serious about quitting my job, but my parents thought I was being facetious.
A. frivolous B. solemn
C. grave D. foolish
15. The High Court quashed a defamation case against a senior IPS officer.
A. indicted B. convicted
C. repealed D. solicited
16. The Indian Premier League seems immune to extraneous factors.
A. relevant B. monetary
C. germane D. external
17. Choose the grammatically correct sentence.
A. Sachin is talented and dedicated than Vinod. B. I prefer tea than coffee.
C. Students find English more easier than
D. I like him more than you.
Mathematics.
18. Complete the sentence by filling in the blanks with the correct tenses from the given options.

It ______ all day yesterday before the big tree in the front yard fell.
A. rained B. had rained
C. has been raining D. was raining
19. Fill in the blank with the correct phrase:

He will _________________ if you point fingers at him.


A. send you to Coventry B. send you to Keresley
C. send you to Hockley D. send you to Sedgley
20. Fill in the blank with the correct phrase:

The phrase ‘a spanner in the works’ refers to a person who ____________.


A. wants to be the centre of attention B. is the top performer
C. does not allow something to go as planned D. does not want to attend a party
21. In the following question, a word has been used in four different ways. Identify the option in which the
usage of the word is incorrect.

LOOK
A. Please look after your sister, as she is not B. The rich snob always looked down on the poor.
www.byjusexamprep.com

keeping well.
D. If you do not want to take part in the tournament,
C. I went to the store to look for a new mobile phone.
you can look forward to it for now.
22. Fill in the blank with the correct phrase from the given options.

This food is as good as we get at a ________.


A. greasy plate B. greasy spoon
C. greasy wheel D. greasy cart
23. Which one of the following is the closest in meaning to the phrase ‘grist to the mill’?
A. Hindrance B. Impediment
C. Useful D. Useless
24. Which of the following figures of speech has not been used in the following sentence?

O wild West Wind, thou breath of Autumn's being,


A. Alliteration B. Metaphor
C. Pun D. Personification
25. Which one of the following figures of speech has not been used in the following?

‘I can eat a horse.’


A. Irony B. Oxymoron
C. Simile D. Hyperbole
26. Direction: Read the following sets of words, and find the odd one out.

Question
A. Army B. Tower
C. Band D. Pacific
27. Question
A. Net B. Baseline
C. Tarpaulin D. Love
28. Question
A. Ship B. Tanker
C. Ballistic D. Jet
www.byjusexamprep.com

29. Question
A. Corpse B. Cadaver
C. Carcass D. Flesh
30. Question
A. Introspection B. Apparition
C. Contemplation D. Musing
31. Direction: The given paragraph consists of five blanks which need to be filled with some words. You are
required to read the paragraph and mark the correct answer as per the context.

Another month, another virus. ____(31)___ does some of the world appear to have made an uneasy
peace with Covid-19, then along comes monkeypox, an unpleasant but, for most people, not fatal
blistering disease caused by a virus that is common in parts of the west and central Africa, but has only
recently made its way in significant numbers to regions where it is not ____(32)___. It has been recorded
in at least 20 countries in the past month, from the Americas to Europe and Australia, ___(33)___ cases
recently confirmed in Wales and Scotland, as well as England. If it continues to spread, it will put yet more
pressure on hard-pressed health services. Efficient management in the community is vital, with people
___(34)___ to come forward if they believe they are infected. Language has an important role to play, with
the UN’s Aid agency warning last month that racist and homophobic ___(35)___ risks stigmatising the
disease and undermining the response.

Which of the following should be used to fill blank 31?


A. no-brainer B. no sooner
C. more so than ever D. ever since
32. Which of the following words should be used to fill blank 32?
A. pandemic B. endemic
C. fervent D. famished
33. Which of the following should be used to fill blank 33?
A. for B. to
C. with D. from
34. Which of the following should be used to fill blank 34?
A. wary B. circumspect
C. terrified D. unafraid
www.byjusexamprep.com

35. Which of the following should be used to fill blank 35?


A. mumbo jumbo B. soliloquy
C. rhetoric D. treatise
36. Direction: The given paragraph consists of five blanks which need to be filled with some words. You are
required to read the paragraph and mark the correct answer as per the context.

In 2000, Jeane Newmaker took her adopted 10-year-old daughter Candace to ___(36)___ ‘attachment
therapy’ retreat designed to increase their emotional bond. While there, Candace underwent an
intervention that’s supposed to ___(37)___ the birthing process. Therapists wrapped her in a flannel sheet
and covered her with pillows to simulate a womb or birth canal. Then they instructed her to fight her way
___(38)___ while four adults (weighing nearly 700 lb in total) tried to stop her. Candace complained and
screamed for help and air, unable to escape from the sheet. After 70 minutes of struggling, pleading that
she was dying, and vomiting and excreting inside the sheet, Candace died ___(39)___ suffocation. This
tragic case highlights an important but often overlooked aspect of ___(40)___ interventions designed to
help people—sometimes they can be harmful or even fatal.

What of the following words should be filled in blank 36?


A. a B. an
C. the D. no article
37. Which of the following words should be used to fill blank 37?
A. inculcate B. replicate
C. eradicate D. enunciate
38. Which of the following words should be used to fill blank 38?
A. out B. in
C. from D. for
39. Which of the following words should be used to fill blank 39?
A. of B. from
C. by D. with
40. Which of the following words should be used to fill blank 40?
A. physiological B. psychological
C. neurological D. biological
41. Direction: A sentence divided into three parts, I, II, and III, is given. Determine the part(s) which require(s)
www.byjusexamprep.com

correction, and mark it as your answer.

It is crucial to point out that our nation is in a (I) / huge financial crisis, however, our economical situation
(II) / is strong, as per the Union Government. (III)
A. Only I B. Both I and II
C. Only II D. Both I and III
42. If developed world’s policies are anything to go by,(I)/ policies can be founded, and the policymakers (II) /
overlooked all of them to date with no likely change in their tactic expected in the upcoming time. (III)
A. Only II B. Only I
C. Both II and I D. Both II and III
43. Being a strong-headed woman, Rinky(I) / stood along with her thoughts and ideologies (II) / and will
continue to work for the people. (III)
A. Both I and III B. Both I and II
C. Only II D. Both II and III
44. Most of the colleges project themselves as not-for-profit even though they are not (I) / but our government
is not admitting the issue, and it is still formulating (II) / policies considering they as not-for-profit ones. (III)
A. Only III B. Both I and III
C. Both II and III D. Only I
45. O account of surge in inflation rates in recent times (I) / Centre has directed the states to do away the
monthly (II) / publication of inflation data and start working on publishing data weekly. (III)
A. Both II and III B. Only II
C. Only III D. Only I
46. In accordance to the availability of limited seats in (I) / the B.Tech course, IIT-Bombay’s authority decided
to give (II) / admission only to the candidates having 90% marks and above. (III)
A. Only III B. Both I and II
C. Both II and III D. Only I
47. Direction: Read the passage given below and answer the questions that follow:

What good would a gathering of literary types be if it did not coincide with a little acrimony and rancour?
South Asia's largest book festival is under way in Jaipur, Rajasthan, a five-hour drive (if you're lucky) from
Delhi. From January 21st to the 25th, a couple of hundred authors, tens of thousands of book-lovers, and
a few Nobel laureates cram the lawns of the Diggi Palace in the Pink City.
www.byjusexamprep.com

The annual Jaipur Literature Festival is now big enough—32,000 attended last year; this year, the tally will
be much higher—that there should be no need for anyone to stir up controversy to get attention.
Nonetheless, shortly before the event, Hartosh Singh Bal, an (Indian) editor of a local magazine, accused
William Dalrymple, a (British) writer who co-directs the festival, of being "pompous" and setting himself up
as an arbiter of writers' taste in the country.

Stung, Mr Dalrymple accused Mr Bal, in turn, of racism. A flurry of angry commentary has followed in the
Indian press and beyond, along with a discussion of whether or why Indian writers crave foreign approval,
especially from Brits.

The festival itself has featured another brewing controversy. Some of the delegates, including Orhan
Pamuk, a Turkish Nobel laureate, and Kiran Desai, an Indian Booker prize winner, had plans to travel
afterwards to another literary event, the Galle Festival, in Sri Lanka. Claiming visa problems, a few
suddenly announced they were pulling out. But the snag may be more political than bureaucratic. Sri
Lanka's government rules with a heavy hand, especially over the Tamil minority, and several journalists
and human-rights activists have been killed or attacked in recent years. The writers had been criticised by
activists for considering a visit to Sri Lanka at all.

Back in Jaipur, the event appears to go from success to success. The festival takes place in the heart of
the city and remains free to all, which matters if locals, including schoolchildren, are to be involved. But as
the crowds and programme grow, the venue is straining to its breaking point, and the nature of the event is
changing. Those who experienced the intimacy of the earliest years lament that it is now impossible to
have conversations with their favourite writers.

The authors, too, may bemoan the festival's increasingly unwieldy size. Junot Diaz, a witty and thoughtful
commentator on the migrants in America, used one session to blame capitalism for encouraging writers to
pursue their work not because they have something important to say but for the sake of getting approval
from the largest audience possible. ‘We know that we need less applause and more conversations,’ he
told a packed room. Promptly—inevitably—the audience clapped.

The passage primarily intends to:


A. Show the negative side of the Jaipur Literature B. Dissuade people from visiting the Jaipur Literature
Festival Festival
C. Show what goes behind a vast event like the D. Inform about the Jaipur Literature Festival, which
Jaipur Literature Festival is mired in controversies.
www.byjusexamprep.com

48. Which of the following is not true about the Jaipur Literature Festival?
A. William Dalrymple is a co-director of the Jaipur B. Jaipur Literature Festival is the largest book
Literature Festival. festival in South Asia.
C. Hartosh Singh Bal is one of the writers who visits D. The festival is conducted in the gardens of the
the Jaipur Literature Festival every year. Diggi Palace in Jaipur.
49. Why would authors lament the increasing size of the Jaipur Literature Festival?
A. Because it may divert writers towards writing just
B. Because it becomes difficult to have a meaningful
to seek approval from a huge audience instead of
conversation with the audience
expressing something important
C. Both A and B D. None of these
50. Which of the following inferences can be drawn about the controversies related to the Jaipur Literature
Festival?
A. Some of the delegates of the Jaipur Literature B. Hartosh Singh Bal did not consider William
Festival found Sri Lanka's literary event more Dalrymple worthy of being associated with the Jaipur
compelling. Literature Festival.
C. The activists did not want the writers visiting the D. The dispute between Bal and Dalrymple
Jaipur Literature Festival to also visit their concluded that Indian writers have lower self-esteem
neighbour's events. as compared to foreign writers.
51. Who among the following has been appointed as the new chairperson of Cellular Operators Association of
India (COAI) for 2022-23?
A. Ajai Puri B. P Balaji
C. Amit Bhatia D. Ajay Aggarwal
E. Pramod K Mittal
52. 'Ennum Ezhuthum' scheme, launched by which state government to fill learning gap which was caused
due to the COVID19 pandemic?
A. Kerala B. Andhra Pradesh
C. Tamil Nadu D. Telangana
E. Karnataka
53. Which of the following airport has become the first airport of India to launch a Vertical Axis Wind Turbine
(VAWT) & Solar PV hybrid (Solar Mill)?
A. Cochin International Airport B. Indira Gandhi International Airport
www.byjusexamprep.com

C. Sardar Vallabhbhai Patel International Airport D. Visakhapatnam International Airport


E. Chhatrapati Shivaji Maharaj International Airport
54. What is the rank of India in Global Skills Report (GSR) 2022, which was released by the Coursera?
A. 65th B. 30th
C. 68th D. 52nd
E. 19th
55. Who among the following has been appointed as a Chief Justice of Delhi High Court?
A. Vipin Sanghi B. SS Shinde
C. Amjad A Sayed D. Ujjal Bhuyan
E. Satish Chandra Sharma
56. To boost the export of Mangoes, the Agricultural and Processed Food Products Export Development
Authority (APEDA) has organized an eight-day long mango festival in which country?
A. Qatar B. Kuwait
C. United Arab Emirates D. Bangladesh
E. Bahrain
57. Which of the following Online service provider companies has invested $5,00,000 with UN Women to
create employment for women?
A. Microsoft B. Instagram
C. Twitter D. Google
E. LinkedIn
58. Who among the following is not among the four prrsons, who have been appointed by the Government to
the Reserve Bank of India (RBI) Board, recently?
A. Anand Gopal Mahindra B. Venu Srinivasan
C. Pankaj Ramanbhai Patel D. Suresh Krishna
E. Ravindra Dholakia
59. Who among the following has become the first woman to lead the White House Office of Science and
Technology Policy?
A. Regina E. Dugan B. Mary Barra
C. Arati Prabhakar D. Mary Chiovitti
E. Noyonika Chakravarty
60. What is the unemployment rate of India in 2020-21, as per the Periodic Labour Force Survey (PLFS),
www.byjusexamprep.com

which was released by the Ministry of Statistics and Programme?


A. 4.2% B. 4.6%
C. 4.8% D. 5.0%
E. 4.0%
61. India’s first private train service under Indian Railways’ ‘Bharat Gaurav Scheme’ has been flagged off
between which two stations?
A. New Delhi and Dharmshala B. Coimbatore and Shirdi
C. Pune to Ahmedabad D. Vasco-da-Gama and Mangluru
E. Thiruvananthapuram and Shirdi
62. Former High Court judge Bhimanagouda Sanganagouda Patil has been appointed as the Lokayukta of
which state?
A. Gujarat B. Madhya Pradesh
C. Uttar Pradesh D. Karnataka
E. Tamil Nadu
63. Which country topped in the World Competitiveness Index 2022 compiled by the Institute for Management
Development?
A. Switzerland B. Singapore
C. Denmark D. Sweden
E. New Zealand
64. Aneesh Gowda is associated with which sports who won the highest number (six gold medals) of
medals in the 4th Khelo India Youth Games 2021?
A. Gymnastic B. Swimming
C. Archery D. Wrestling
E. Table tennis
65. The government of India has introduced Agnipath Military recruitment scheme to recruit defense troops for
how many years of short duration?
A. 4 years B. 5 years
C. 6 years D. 7 years
E. 8 years
66. The central govt has decided to distribute fortified rice in how many aspirational and high-burden districts
across the country through Public Distribution System in 2022-23?
www.byjusexamprep.com

A. 112 B. 175
C. 200 D. 291
E. 301
67. Which of the following has adopted an India-sponsored resolution on multilingualism that mentions the
Hindi language for the first time?
A. WTO B. UNGA
C. NATO D. UNESCO
E. WMO
68. NHAI enters Guinness World Record for laying 75 km highway in 105 hours and 33 minutes between
Amravati and Akola districts. It is a part of which of the following national highways?
A. NH 53 B. NH 42
C. NH 8 D. NH 12
E. NH 18
69. Second edition of National e-Governance Service Delivery Assessment 2021 (NeSDA 2021) covers
services across __________________.
A. 5 sectors B. 6 sectors
C. 7 sectors D. 8 sectors
E. 9 sectors
70. Who has been appointed as the ombudsman under Mahatma Gandhi National Rural Employment
Guarantee Scheme (MGNREGA)?
A. N J Ojha B. Raghuvansh Prasad Singh
C. Rohit Kumar D. Shashi Bhushan Kumar
E. Nagendra Nath Sinha
71. Which of the following Indian has been recognized as Global Sustainable Development Goal (SDG)
pioneer by UNGC?
A. Vishal V. Sharma B. Vinay Mohan Kwatra.
C. Ramkrishna Mukkavilli D. Srinivas Das
E. Kiran Mazumdar Shaw
72. Which of the following has been awarded with the Regional Director Special Recognition Award by WHO?
A. Resource Centre for Tobacco Control, PGIMER B. National Tobacco Control Cell, AIMS
C. Jawaharlal Nehru Cancer Hospital & Research D. National Institute of Mental Health and
www.byjusexamprep.com

Centre Neurosciences
E. None of the above
73. After defeating which country, India has won the final of the inaugural FIH Hockey 5s championship in
Lausanne in Switzerland?
A. Iceland B. Canada
C. Bangladesh D. Malaysia
E. Poland
74. Which of the following coastal state of India has launched a 'Beach Vigil App' to ensure the management
of beaches and the security of tourists?
A. Kerala B. Goa
C. Odisha D. Tamil Nadu
E. Karnataka
75. Which of the following Indian institutes have been ranked 155th in the QS World University Ranking 2023?
A. Indian Institute of Science, Bangalore B. Indian Institutes of Technology, Bombay
C. Indian Institutes of Technology, Delhi D. Indian Institutes of Technology, Kharagpur
E. Indian Institutes of Technology, Madras
76. Who among the following has been appointed as a Director of the Asia and Pacific Department (APD),
International Monetary Fund?
A. Changyong Rhee B. Sanjaya Panth
C. Uma Ramakrishnan D. Jeromin Zettelmeyer
E. Krishna Srinivasan
77. Who among the following has been elected as the new president of Albania?
A. Ilir Meta B. Edi Rama
C. Bujar Nishani D. Pandeli Majko
E. Bajram Begaj
78. Which of the following state's Anti-Corruption Bureau has launched the ‘14400 app’ to report corrupt
government officials in the state?
A. Uttar Pradesh B. Andhra Pradesh
C. Punjab D. Madhya Pradesh
E. Tamil Nadu
79. Who among the following would receive the lifetime achievement award at MTV Movie & TV Award?
www.byjusexamprep.com

A. Jennifer Aniston B. Lynda Lopez


C. Jennifer Garner D. Jennifer Lopez
E. Jennifer Lawrence
80. Who among the following has won the Best Actor (Female) award at the 22nd International Indian Film
Academy Awards in Abu Dhabi, United Arab Emirates?
A. Sanya Malhotra B. Kirti Sanon
C. Kiara Advani D. Vidya Balan
E. Taapsee Pannu
81. Direction: Given below is a statement followed by two conclusions I and II. You are required to read the
statement and check whether the conclusions follow or not.

1) Statement:

Emerging market economies, including India, need to keep foreign exchange reserves liquid after building
a substantial buffer.

Conclusions:

I. In extreme cases like the global financial crisis or the Covid-19 outbreak, there could be a requirement
of liquid reserves.

II. Liquid cash would result in strong bargaining power for us while importing essential goods.
A. Only I follows B. Only II follows
C. Both follow D. None follows
82. Statement:

Almost a year after barring Mastercard Asia/Pacific Pte Ltd from onboarding new domestic customers on
its network, the Reserve Bank of India (RBI), on Thursday, lifted business restrictions imposed on the
global card player ‘with immediate effect’.

Conclusions:

I. The lifting of the ban on Master Card would surely increase competition in the debit/credit card business
in India.
www.byjusexamprep.com

II. Master Card’s re-entry into the Indian market would spice up the financial instruments business.
A. Only I follows B. Only II follows
C. Both follow D. None follows
83. Statement:

From Frankfurt and New York to Istanbul and Beijing, skygazers could enjoy a beautiful supermoon rising
over the horizon in an impressive astronomic spectacle this weekend.

Conclusion:

I. Supermoon would attract large crowds to places where there is a clear view of the sky.

II. In major cities around the world, citizens would line up outside of lunar observatories to witness the
impressive astronomic spectacle.
A. Only I follows B. Only II follows
C. Both follow D. None follows
84. Statement:

Archival data derived from the Hubble Space Technology and other NASA observatories has unveiled that
a white dwarf star is syphoning off debris from both the inner and outer corners of the entire planetary
system.

Conclusions:

I. The characteristics of a white dwarf include its penchant to draw off debris from a planetary system.

II. White Dwarf planets are the next in line of new generation planets that have been witnessed in recent
times.
A. Only I follows B. Only II follows
C. Both follow D. None follows
85. Statement:

While India was declared polio-free eight years ago by the World Health Organisation, traces of the virus
have been found periodically in sewage samples from time to time, without leading to anything serious.

Conclusion:
www.byjusexamprep.com

I. There is no cause for alarmist hysteria as , the situation is not serious.

II. India is yet to achieve polio-free status in the actual sense of the term
A. Only I follows B. Only II follows
C. Both follow D. None follows
86. Direction: In the question below, a statement is given followed by two arguments. Choose the most
appropriate option that tells us whether that particular argument is strong or not.

Statement:

Should all the army professionals be provided life insurance policies from government-aided funds?

Arguments:

I. Yes, it would ensure their family security if they died before or after retirement.

II. No, it will be a total waste of public money and an extra burden on the exchequer.
A. Only argument I is strong. B. Only argument II is strong.
C. Both arguments I and II are strong. D. Neither argument I nor argument II is strong.
87. Statement:

Should people spend more time in a fitness and exercise regimen?

Arguments:

I. Yes,doctors stress on the importance of fitness and exercise for good health

II. Yes, exercise and working out would lead to a reduced risk of health hazards such as heart ailments,
diabetes and obesity.
A. Only argument I is strong. B. Only argument II is strong.
C. Both arguments I and II are strong. D. Neither argument I nor II is strong.
88. Statement:

Should the government introduce more programs and policies for the upliftment of the poorer sections of
society?
www.byjusexamprep.com

Arguments:

I. Yes, because if the government does not help them out, who will??

II. Yes, because a country’s comprehensive development depends on the development of the weaker
sections of society.
A. Only argument I is strong. B. Only argument II is strong.
C. Both arguments I and II are strong. D. Neither argument I nor II is strong.
89. Statement:

Should the Ministry of culture ,on the occasion of International Museum Day, make admissions to all
museums free for a week.

Argument:

I. Yes, because it not only makes our art and culture accessible to all but also provides us with an
opportunity to propagate our cultural heritage.

II. No, because it would lead to a huge loss of money from gate receipts.
A. Only Argument I is strong. B. Only Argument II is strong.
C. Both arguments I and II are strong. D. Neither Argument I nor Argument II is strong.
90. Statement:

The recent threats to data privacy has increased the need for government to come up with strong data
privacy law. So, should the government do it?

Arguments:

I. Yes, because the need to preserve the identities of its citizens will grow as technology progresses.

II. Yes, because it will provide citizens with an assurance that their personal details are safe.
A. Only Argument I is strong. B. Only Argument II is strong.
C. Both arguments I and II are strong. D. Neither Argument I nor Argument II is strong.
91. Taking antibiotics can successfully protect you against bacterial infections, and it is often an important part
of maintaining your health. But where regular consumption of antibiotics was once thought sufficient to
protect you, new research suggests that unhealthy eating habits can reduce the efficacy of antibiotics.
www.byjusexamprep.com

Which of the following can be inferred from the information provided in the question?
B. Our eating habits have a lot to do with how
A. Bacteria have become more averse to antibiotics.
healthy we are.
C. We should stop eating junk food to avoid the risks D. Bacterial infections have also grown over the
of bacterial infections. years, nullifying the effects of antibiotics.
92. A shift favouring more plant-based foods has been steadily gaining momentum worldwide within the
medical and athletic communities. However, some individuals continue to express concern regarding
using plants as protein sources in sports nutrition products.

Which of the following inferences can be drawn from the given information?
A. Sportspersons have different nutritional needs,
B. Plant-based foods have proved their nutritional
compared to other people due to the nature of their
mettle as opposed to animal-based protein.
work.
C. One’s nutritional needs are dependent on the kind D. The concerns surrounding using plant-based
of work one does. protein sources are something to think about.
93. Here’s a big number: $44 trillion. That’s how much of the world’s total economic output is dependent on
animals and ecosystems, according to the World Economic Forum.

Which of the following can be inferred from the given information?


B. Now that we know the economic costs
A. Our bionetwork is just as important for our
surrounding our environment, we need to do
economic prospects as our scientific advancement.
everything in our power to preserve its integrity.
C. Our economy is vastly reliant on our ecosystem D. The total economic output reliant on animals and
for its growth. ecosystems is quite large.
94. Greenhouse gas emissions should be subject to legal controls in the US and phased out under the Toxic
Substances Control Act, according to a group of scientists and former public officials, in a novel approach
to the climate crisis.

Which of the following can be inferred from the given information?


A. It is quite possible that there has been an excess B. Climate crisis is largely dependent on how well we
of greenhouse gases in the US casunig concern. tackle the global emission of greenhouse gases.
C. Greenhouse gases has been a huge thorn in the D. Greenhouse gas emissions subjected to legal
flesh of Toxic substances Control Act. controls can have various repercussions.
www.byjusexamprep.com

95. Polar bears have become the furry face of the climate crisis, with experts suggesting the animals could be
all but extinct in a matter of decades as the Arctic sea ice they hunt from melts away.

Which can be inferred from the given information?


A. Polar bears will become become extinct within the B. Polar bears have found a way to survive the
next decade . climate crisis.
D. Polar bears are likely to be replaced by those
C. Polar bears have been adversely affected by the
species who have a knack to survive in warm
climate crisis.
conditions.
96. Direction: Given below is a statement followed by two assumptions I and II. You are required to go
through the statement and find out which assumptions follow or not.

Statement:

Qatar Airways, one of the Mideast's largest carriers known for on-board comfort and luxury, said its profits
over the past fiscal year topped $1.5 billion, marking the highest ever earnings for the state-owned carrier
as it prepares to see a record surge in travellers for the upcoming FIFA World Cup soccer games.

Assumptions:

I. FIFA World Cup s brings expectations of higher number of travellers .

II. Qatar Airways is looking to expand to other territories to book more profits.
A. Only I follows B. Only II follows
C. Both follow D. None follows
97. Statement:

An International Monetary Fund (IMF) working paper published in May ranked India 127th among 162
countries in terms of speed of road transport. A comparison of the rankings shows that even Pakistan is
ranked higher than India in terms of average speed. The paper suggested improving the road conditions
and improving traffic regulations.

Assumptions:

I. Improving road conditions would improve the speed of road transport


www.byjusexamprep.com

II. India is expected to be better than Pakistan in terms of average speed/


A. Only I follows B. Only II follows
C. Both follow D. None follows
98. Statement:

Media reports revealed that the tests of at least three weapons- the ASTRA-1 and ASTRA-2 beyond visual
range air-to-air missiles (BVRAAMs) and the new generation anti-radiation missile (NGARM) Rudram-1-
were slated to happen last month. While the first two have garnered their fair share of attention, the
Rudram missile has remained elusive.

Assumptions:

I. There is no information about the Rudram missile.

II. The tests of three weapons have been kept under wraps for much of their development stage.
A. Only I follows B. Only II follows
C. Both follow D. None follows
99. Statement:

A severe drought has been affecting most of Afghanistan since early 2021 . It has crippled crop production
and livestock leading to critical food shortages. Nearly 20 million Afghans are deprived of food currently,
as per reliefweb.int.

Assumptions:

I. Afghanistan has been facing drought-like conditions for much of 2021 .

II. Poor water management can largely be attributed to the water crisis in Afghanistan.
A. Only I follows B. Only II follows
C. Both follow D. None follows
100. Statement:

With machine learning, artificial intelligence, quantum computing and other high-tech advances, most of
us have wondered if our jobs will be taken over by technology. The bad news is, this is already happening
for some professions doing rote work, such as telemarketers.
www.byjusexamprep.com

Assumptions:

I. High tech advances is the cause of job losses in some professions

II. Technological changes are a threat to the economy


A. Only I follows B. Only II follows
C. Both follow D. None follows
101. Almost one in three people in England are obese. Three decades ago, it was only about one in seven.
Data from Northern Ireland, Scotland and Wales follows similar patterns. Expanding waistlines can
shorten lives. Obesity is the second biggest preventable cause of cancer in the UK – behind smoking –
and an important factor in heart disease and type 2 diabetes. Covid hits harder because obesity has
become too common. Boris Johnson’s government promised action to tackle this worsening public health
situation – action that would reduce pressure on the NHS. The independent food strategy produced for
ministers last year by the businessman Henry Dimbleby had a remit that went beyond the issues
surrounding obesity, poverty and junk food.

Which of the following observations is best supported in the passage?


B. Obesity is just as responsible for cancer as
A. Obesity is a widely prevalent ailment in England.
smoking.
C. Carelessness is increasing the he cases of D. COVID-19 pandemic had nothing to do with
obesity in people. obesity.
102. The decision by Priti Patel, the home secretary, to extradite the WikiLeaks founder Julian Assange to the
US ought to worry anyone who cares about journalism and democracy. Mr Assange, 50, has been
charged under the US Espionage Act, including publishing classified material. He faces up to 175 years in
jail if found guilty by a US court. This action potentially opens the door for journalists anywhere in the
world to be extradited to the US for exposing information deemed classified by Washington.

The paragraph best supports the statement that:


A. Anyone who goes around misusing the powers of B. People in the US involved in leaking classified
journalism will be deported to the US. information will find it difficult to escape punishment .
C. Anyone who leaks information marked classified D. The US has enormous power and it can leverage
by the US will not face the music. the same to bend rules its way.
103. In the hours following Moscow’s invasion of Ukraine on 24 February, every moment that Kyiv could hold
out was regarded as a victory. Many – not only in the Kremlin – expected it to fall within days. Vladimir
www.byjusexamprep.com

Putin’s hubris, Russian military shortcomings, courageous Ukrainian resistance and the sight of western
nations rushing to show support created a glimmer of hope. Three and a half months on, the picture has
darkened again. After Mariupol, Bucha and others, the eastern city of Sievierodonetsk is the latest scene
of civilian suffering.

The passage best supports the statement that:


A. Ukraine’s resistance is not likely to sustain much B. Russia’s persistent attacks on Ukraine has left it
longer. unduanted.
C. Ukraine’s strong resistance was expected to D. The western nations have not failed to deliver a
continue. killer blow to Russia.
104. OVERDOSE: PRESCRIPTION ::
A. deprivation: materialism B. indiscretion :convention
C. ailment: medicine D. adventure: voyage
105. FRESCO:WALL::
A. fountain : courtyard B. parquetry:floor
C. hay:roof D. statuary: passage
106. Principle 1 – When two or more persons agree to do, or cause to be done, an illegal act, or an act which is
not illegal, by illegal means, such an agreement is designated a criminal conspiracy.

Principle 2 - No agreement except an agreement to commit an offence shall amount to a criminal


conspiracy unless some act besides the agreement is done by one or more parties to such agreement in
pursuance thereof.

Facts – A and B entered into an agreement to beat C. The agreement -


A. Is not criminal conspiracy. B. Amounts to illegal act.
C. Amounts to legal act but by illegal means. D. All of the above
107. Principle 1 – When two or more persons agree to do, or cause to be done, an illegal act, or an act which is
not illegal, by illegal means, such an agreement is designated a criminal conspiracy.

Principle 2 - No agreement except an agreement to commit an offence shall amount to a criminal


conspiracy unless some act besides the agreement is done by one or more parties to such agreement in
pursuance thereof.
www.byjusexamprep.com

Facts – A and B decided to pay off C’s debt of Rs.10,00,000. For that purpose they tend to rob a bank. A
and B bought a gun and masks for the purpose of bank robbery.
A. This is criminal conspiracy within the meaning of B. This is not criminal conspiracy within the meaning
the above principle. of the above principle.
C. This amounts to do illegal act. D. All of the above
108. Principle – The provisions of Indian Penal Code shall apply to any offence committed by any citizen of
India in any place without and beyond India or any person on any ship or aircraft registered in India
wherever it may be.

Facts – A who is a citizen of Afghanistan beat B who is citizen of India on Indian ship which is in Pacific
Ocean when A beat B. Here –
A. A is liable under Indian Penal Code. B. A is not liable under Indian Penal Code.
C. A cannot be tried in India. D. Both b and c
109. Principle – The provisions of Indian Penal Code shall apply to any offence committed by any citizen of
India in any place without and beyond India or any person on any ship or aircraft registered in India
wherever it may be.

Facts – A who is a citizen of Afghanistan beat B who is citizen of India in Afghanistan. Here –
A. A is liable under Indian Penal Code. B. A is not liable under Indian Penal Code.
C. A can be tried in India. D. Both a and c
110. Principle – Whenever any person is convicted of an offence for which under this Code the Court has
power to sentence him to rigorous imprisonment, the Court may, by its sentence, order that the offender
shall be kept in solitary confinement for any portion or portions of the imprisonment to which he is
sentenced, not exceeding three months in the whole.

Facts – An accused was convicted for an offence punishable with simple imprisonment upto 3 years.
However the court convicted the accused for 2 years simple imprisonment and 2 months for solitary
confinement. The decision of court is –
A. Valid B. Executable
C. Invalid D. Both a and b
111. Principle – Nothing is an offence which is done by a person who is, or who by reason of a mistake of fact
and not by reason of a mistake of law, in good faith believes himself to be, bound by law to do it.
www.byjusexamprep.com

Facts – A, a police officer, wrongfully arrested B who has committed a non-cognizable offence (i.e., police
cannot arrest without order of magistrate) without warrant. Can A claim exemption under the above
principle?
A. Yes, as the mistake by A is that of law. B. Yes, as mistake by A is that of fact.
C. No, as mistake by A is that of law. D. No, as mistake by A is that of fact.
112. Principle – Nothing is an offence which is done by a person who is, or who by reason of a mistake of fact
and not by reason of a mistake of law, in good faith believes himself to be, bound by law to do it.

Facts – A, a police officer, wrongfully arrested B thinking him C, in good faith. Can A claim exemption
under the above principle?
A. Yes, as the mistake by A is that of law. B. Yes, as mistake by A is that of fact.
C. No, as mistake by A is that of law. D. No, as mistake by A is that of fact.
113. Principle – Whoever abets the commission of an offence shall, if the person abetted does the act with a
different intention or knowledge from that of the abettor, be punished with the punishment provided for the
offence which would have been committed if the act had been done with the intention or knowledge of the
abettor and with no other.

Facts – A instigates B to burn Z’s house. B sets fire to the house and at the same time commits theft of
property of Z. Here A is guilty of -
A. Theft B. Abetment to theft and burning of house
C. Abetment to burning of house D. Abetment to theft
114. Principle – Whoever abets the commission of an offence shall, if the person abetted does the act with a
different intention or knowledge from that of the abettor, be punished with the punishment provided for the
offence which would have been committed if the act had been done with the intention or knowledge of the
abettor and with no other.

Facts – A instigates a child of 7 years to put poison into the food of Z, and gives him poison for that
purpose. The child, in consequence of the instigation, by mistake puts the poison into the food of Y, which
is by the side of that of Z. Here -
A. A has committed no offence. B. A is liable for murder.
C. A is liable for abetment to murder. D. None of the above
115. Principle - Culpable homicide is not murder if the offender, whilst deprived of the power of self-control by
grave and sudden provocation, causes the death of the person who gave the provocation or causes the
www.byjusexamprep.com

death of any other person by mistake or accident.

Facts – A was beating his wife with a lathi, which accidently fell on the child in arms of A’s wife. Discuss
the liability of A.
A. Liable for culpable homicide. B. Liable for murder.
C. Both a and c D. None of the above
116. Principle 1 - Culpable homicide is not murder if the offender, whilst deprived of the power of self-control by
grave and sudden provocation, causes the death of the person who gave the provocation or causes the
death of any other person by mistake or accident.

Principle 2 – Provided the provocation is not given by anything done in obedience to the law, or by a public
servant in the lawful exercise of the powers of such public servant.

Facts - A is lawfully arrested by a police officer. A is excited to sudden and violent passion by the arrest,
kills the police officer. This is -
A. Murder B. Culpable homicide
C. Culpable homicide not amounting to murder. D. Both b and c
117. Principle – Whoever takes or entices any minor under sixteen years of age if a male, or under eighteen
years of age if a female, or any person of unsound mind, out of the keeping of the lawful guardian of such
minor or person of unsound mind, without the consent of such guardian, is said to kidnap such minor or
person from lawful guardianship.

Facts – A kidnaps A, a child of 13 years of age with his consent after giving him sweets. Does this amount
to kidnapping?
A. Yes, as A enticed the child. B. No, as A obtained the consent of child.
C. Yes, as A did not obtain consent of guardian. D. Both a and c
118. Principle - A person to whom money has been paid, or anything delivered, by mistake or under coercion,
must repay or return it.

Facts – A and B jointly owe Rs.1000 to C, A pays the whole amount to C. B, not knowing this, pays
Rs.1000 again to C. Here -
B. C can retain the amount received from A as well
A. C is bound to repay the amount to B
as B
C. There is no coercion practiced on B, thus, C can D. Both b and c
www.byjusexamprep.com

retain the amount.


119. Principle – If a person, incapable of entering into a contract, or any one whom he is legally bound to
support, is supplied by another person with necessaries suited to his condition in life, the person who has
furnished such supplies is entitled to be reimbursed from the property of such incapable person.

Facts – A who is bound to support B, supplied B (her niece) with a number of beautiful dresses. Can A
recover the price of the dresses from B?
A. Yes, as A is bound to support B B. Yes, as A supplied with necessaries.
C. No D. Both a and b
120. Principle 1 - When the person to whom the proposal is made signifies his assent thereto, the proposal is
said to be accepted. A proposal, when accepted, becomes a promise.

Principle 2 - Every promise and every set of promises, forming the consideration for each other, is an
Agreement.

Facts – A send his servant to find his lost pet dog. Thereafter, A announced a reward of Rs.10000,
whoever finds his pet dog. The servant found the dog and claimed reward.
A. The servant cannot claim reward as there was no B. Finding the dog amounts to acceptance of offer
communication of offer to him. and thus a concluded contract.
C. The servant can claim reward. D. Both b and c
121. Principle 1 – Coercion is the committing, or threatening to commit, any act forbidden by the Indian Penal
Code or the unlawful detaining, or threatening to detain, any property, to the prejudice of any person
whatever, with the intention of causing any person to enter into an agreement.

Principle 2 - When consent to an agreement is caused by coercion, fraud or misrepresentation, the


agreement is a contract voidable at the option of the party whose consent was so caused.

Facts – A threatened B to sign a document, otherwise B will suffer a divine displeasure.


A. the contract is entered under coercion by B and
B. the contract is voidable at the instance of A
thus, voidable
C. the contract is voidable at the instance of B D. the contract is not voidable.
122. Principle 1 - All agreements are contracts if they are made by the free consent of parties competent to
contract.
www.byjusexamprep.com

Principle 2 – A contract which is concluded without free consent is voidable at the instance of the party
whose consent was so obtained.

Facts – A contract is made between a minor(incompetent to contract) to sell his pen to A. A, considering
the minor as of competent age due to his tall stature agreed to but the pen. Is the contract valid?
B. Yes, as the age purchaser is relevant and not of
A. No, as the minor is not competent to contract.
the buyer.
C. Yes, as the minor intentionally deceived the D. Yes, as the contract is voidable at the instance of
purchaser. purchaser.
123. Principle 1 – The communication of a revocation is complete as against the person who makes it, when it
is put into a course of transmission to the person to whom it is made, so as to be out of the power of the
person who makes it.

Principle 2 - The communication of a revocation is complete as against the person to whom it is made,
when it comes to his knowledge.

Facts – A revokes his proposal which was made to B by post. The revocation is complete as against A –
A. When B receives the post. B. When the post is dispatched.
C. When the agent of B receives post. D. Both a and c
124. Principle 1 – A contract is said to be induced by “undue influence” where the relations subsisting between
the parties are such that one of the parties is in a position to dominate the will of the other and uses that
position to obtain an unfair advantage over the other.

Principle 2 – When consent to an agreement is caused by coercion, fraud or misrepresentation, the


agreement is a contract voidable at the option of the party whose consent was so caused.

Facts - A, being in debt to B, the money-lender of his village, contracts a fresh loan on terms which appear
to be unconscionable.
A. The contract is valid B. The contract is void
C. The contract is voidable. D. The contract is illegal.
125. Principle – A proposal can be revoked by the death or insanity of the proposer, if the fact of his death or
insanity comes to the knowledge of the acceptor before acceptance.
www.byjusexamprep.com

Facts – A proposes to sell her land to B for Rs.100000. Subsequently, A goes mad, after B had accepted
the proposal. Is this a valid contract?
A. No, as A had gone mad. B. Yes, the acceptance has been validly made.
C. There is no valid acceptance, as one of the
D. Both a and c
parties has not remained sane.
126. Principle – A proposal can be revoked by the death or insanity of the proposer, if the fact of his death or
insanity comes to the knowledge of the acceptor before acceptance.

Facts – A proposes to sell land to B for Rs.100000. Subsequently, A goes mad, before B could accept the
proposal. Is this a valid contract?
A. No, as A had gone mad. B. Yes, the acceptance has been validly made.
C. The contract has been validly revoked. D. Both a and c
127. Principle – Every promise and every set of promises, forming the consideration for each other, is an
agreement.

Facts – A promises to pay B Rs. 1000. This is –


A. A promise B. An agreement
C. A contract D. None of the above
128. Principle - The principle of volenti non fit injuria states that the defendant is not liable when the plaintiff
consents to the act which has caused injury to him.

Facts – A is watching cricket match in Delhi stadium. When he was busy in his phone, a ball hit his head,
leading to severe injury. Here –
A. A can claim compensation from the cricket match
B. A can claim compensation from the batsman.
organisers.
C. A cannot claim the compensation. D. Either A or B
129. Principle - The principle of volenti non fit injuria states that the defendant is not liable when the plaintiff
consents to the act which has caused injury to him.

Facts – Cricket players are practicing in a field covered by net.. During the practice, the ball went outside
the field from a small hole in the net, and hit a passerby X. X was injured. Here –
B. The principle of volenti non fit injuria has no
A. The cricket players are liable.
application here.
www.byjusexamprep.com

C. X has consented to the risk, as he knew he might D. Both a and b


get hurt due to presence of cricket field nearby.
130. Principle – The principle of volenti non fit injuria states that the defendant is not liable when the plaintiff
consents to the act which has caused injury to him.

Facts - A house is being constructed on a busy road. During the contraction, a brick hit a passerby B, and
he was seriously injured. Here the doctrine –
A. Volenti non fit injuria will apply. B. Volenti non fit injuria will not apply.
C. The workers have no liability. D. Both b and c
131. Principle 1 - Trespass means the wrongful disturbance of possession of land or goods of another person.

Principle 2 - A person who intentionally and without consent enters another person’s property is a
trespasser.

Facts – A invites his friend B for a dinner party at his house. They argued at a point and A asked B to
leave. B did not leave. Can A sue B for trespass?
A. Yes, as the invite turned into trespass later on. B. No, as A himself invited B
C. No, as A and B were equally at fault. D. b and c
132. Principle 1 - Trespass means the wrongful disturbance of possession of land or goods of another person.

Principle 2 - A person who intentionally and without consent enters another person’s property is a
trespasser.

Facts – A invites his friend B for dinner party at his house. They argued at a point and A sued B for
trespass, can he do so?
A. Yes, as the invite turned into trespass later on. B. No, as A himself invited B
C. No, as A and B were equally at fault. D. b and c
133. Principle 1 - Trespass means the wrongful disturbance of possession of land or goods of another person.

Principle 2 - A person who intentionally and without consent enters another person’s property is a
trespasser.

Facts – A regularly cleans the public path adjacent to his house. B with muddy feet walks over the path
just after A mobbed the path. A sued B for trespass. Decide.
www.byjusexamprep.com

A. A cannot sue B for trespass. B. A can sue B for trespass.


C. Depends on facts of circumstances of each case. D. None of the above
134. Principle – Damnum sine injuria i.e., damage to a person without legal injury to that person is not
actionable in the court of law.

Facts – A is running a primary school. B started a nursery and primary school adjacent to the school of A.
Many children started enrolling themselves in the school of B causing high losses to A. A sued B for
damages. Discuss the liability of B.
B. B is liable for damages as he caused loss of
A. B is not liable for damages to A
revenue to A
C. Damage without legal injury is not actionable. D. Both a and c
135. Principle – Injuria sine damnum i.e., legal injury (violation of right) without damage to a person is
actionable in court of law.

Facts – A stopped B from casting vote in elections as B is opposition of A. However, the party for which B
wanted to vote won. Can B sue A?
A. Yes B. No
C. Depends on facts and circumstances D. None of the above
136. Principle – Res ipsa loquitur, i.e., facts speaks for itself. In such cases, the plaintiff need not proof the
negligence of the defendant when the facts speak for itself.

Facts – A got her abdomen operated. Due the negligence of the surgeon, a pair of scissors was left in her
abdomen. Due to severe stomach pain by A, it was later discovered, and removed by a second surgery.
Here -
B. Surgeon can be held responsible but A will have
A. Surgeon cannot be held responsible because it is
to prove in the court of law that the surgeon was
merely a human error.
grossly negligent.
C. Surgeon will be responsible and A need not prove
surgeon’s negligence because presence of scissors D. None of the above
in her abdomen is sufficient proof.
137. Principle – Negligence is a breach of duty or a failure of one party to exercise the standard of care
required by law, resulting in damage to the party to whom the duty was owed. A plaintiff can take civil
action against the respondent, if the respondent’s negligence causes the plaintiff injury or loss of property.
www.byjusexamprep.com

Facts – A brought alcohol, which contained remains of dead snail. A suffered food poisoning on
consumption of the alcohol and sued the company owner.
A. The company is not liable. B. The company is liable for negligence.
C. The company took reasonable care. D. Both a and c
138. Principle – Negligence is a breach of duty or a failure of one party to exercise the standard of care
required by law, resulting in damage to the party to whom the duty was owed. A plaintiff can take civil
action against the respondent, if the respondent’s negligence causes the plaintiff injury or loss of property.

Facts – A truck driver was driving the truck in his lane. Seeing a car driver coming from opposite direction
he suddenly, changed the lane. The car driver realizing that he is driving in wrong lane changed the lane
at the same time, leading to collision with the truck. Here –
A. The truck driver is liable for negligence. B. The truck driver is not liable for negligence.
C. The truck driver owed no duty towards the person
D. Both b and c
who drives in wrong lane.
139. Principle – A tort is a civil wrong that causes harm to another person by violating a protected right.

Facts – A murdered B. Is this a tort?


A. Yes B. No
C. May be D. None of the above
140. Principle – Whoever by words either spoken or intended to be read, or by signs or by visible
representations, makes or publishes any imputation concerning any imputation concerning any person
intending to harm, or knowing or having reason to believe that such imputation will harm, the reputation of
such person, is said to defame that person.

Facts – In a community there is a custom of stealing shoes of bridegroom during the marriage ceremony.
The shoes of the bridegroom were stolen by A. B announced that A has stolen the shoes. Everyone
present in the marriage party started staring at A. A got really embarrassed. Here –
A. A can sue B for defamation. B. A cannot sue B for defamation.
C. This is not a case of defamation. D. Both b and c
141. Five people (A to E) play five different games on five different days of the same week starting from
Monday. No one plays on Saturday and Sunday. A plays the game before Wednesday but does not play
Boxing. Badminton is played before Friday but not on Monday. Tennis is played after Badminton. Cricket is
played on Tuesday. Archery, which is played by E, is played after Boxing, which is not played by B. Two
www.byjusexamprep.com

people play the game between D and the one, who plays Cricket. C does not play Badminton, which is not
played on Wednesday. Who among the following plays tennis and on which day?

A. C, Wednesday B. D, Friday
C. E, Wednesday D. B, Friday
E. A, Monday
142. Only one term of the following series does not follow the pattern that others follow and, hence, becomes
an odd one out in the group. Find the odd one out.

697989 192939 596989 394959

A. 192939 B. 394959
C. 697989 D. 596989
143. In a certain code language, ‘ABSOLUTE’ is coded as ‘ETABULSO’, and ‘ACCIDENT’ is coded as
‘TNACEDCI’ How is ‘ACCURATE’ coded in that language?
A. CUETACAR B. ETACARCU
C. CARCUETA D. None of the above
144. Which two symbols should be interchanged to make the equation correct?

15 – 2 + 15 ÷ 5 2 = 31
A. + and – B. – and ÷
C. × and + D. – and ×
145. Direction: Study the information given below carefully and answer the questions that follow.

The symbol codes given below denote the relation of one family member with the other.

‘A * B’ means A is the mother of B

‘A + B’ means A is the brother of B

‘A = B’ means A is the wife of B

‘A ^ B’ means A is the son of B

Which of the following represents that T is the husband of Q?


www.byjusexamprep.com

A. Q * V + O ^ T B. V ^ T * Q = O
C. Q * V + T * O D. T + O ^ Q * V
E. None of the above
146. A series is given with one term missing. Select the correct answer from the following options that will
complete the series.

5G, 8I, 13M, 20U, ?


A. 28J B. 30L
C. 29K D. 31M
E. None of the above
147. Direction: Study the information given below carefully and answer the questions that follow.

The symbol codes given below denote the relation of one family member with the other.

‘A * B’ means A is the mother of B

‘A + B’ means A is the brother of B

‘A = B’ means A is the wife of B

‘A ^ B’ means A is the son of B

How is M related to S if M * W = H + S ^ D?
A. Daughter B. Mother-in law
C. Daughter-in law D. Mother
E. None of the above
148. When Aayush shifted by five places towards the right, he became 9th from the left end in a row of 12 boys
. What was the earlier position of Aayush from the right end of the row?
A. 8th B. 9th
C. 5th D. 6th
149. The angle between the minute hand and the hour hand of a clock when the time is 8:20, is:
A. 110⁰ B. 120⁰
C. 130⁰ D. 140⁰
www.byjusexamprep.com

150. Select the option that is related to the first letter-cluster in the same way as the fourth letter-cluster is
related to the third letter-cluster.

AMETHYST : ? :: PURPLE : ELPPUR


A. TSYHAMET B. TSYHTEMA
C. AMETTSYH D. AMTHYSET
www.byjusexamprep.com

Solutions

1. A
Sol. Passage summary
The passage is about Asimov and his works. The passage starts with a description of
Asimov’s Three Laws of Robotics. He wrote several novels and short stories exploring the
interaction between humans and robots. The author states that Asimov’s works demonstrate
his belief in self-determination, human liberty, and freedom. He valued these ideals more as
he was from a country where these were denied.
Genre: Literature
Number of words: 405
Type of question: Main point/ theme based question
The quick reading of the passage will help identify the main point of the passage- the passage
talks about Asimov, his background, his works, and his ideals.
All these are mentioned concisely in option A.
Hence, option A is the correct answer.
2. C
Sol. Passage summary
The passage is about Asimov and his works. The passage starts with a description of
Asimov’s Three Laws of Robotics. He wrote several novels and short stories exploring the
interaction between humans and robots. The author states that Asimov’s works demonstrate
his belief in self-determination, human liberty, and freedom. He valued these ideals more as
he was from a country where these were denied.
Genre: Literature
Number of words: 405
Type of question: Author’s purpose
Let’s read the relevant paragraph. ’In works such as The End of Eternity and The Stars, Like
Dust, Asimov came down firmly on the side of human liberty and the freedoms embodied in
the US Constitution and the Bill of Rights. Isaac Asimov immigrated from a place that did not
www.byjusexamprep.com

cherish human liberty and freedom, which encouraged him, like millions of immigrants before
and since, to embrace those values even more.’
Summing up, we can say that the paragraph talks of Asimov’s support for human liberty and
freedom and how he came to cherish such values.
Option C summarises the points best. Hence, option C is the correct answer.
3. C
Sol. Passage summary
The passage is about Asimov and his works. The passage starts with a description of
Asimov’s Three Laws of Robotics. He wrote several novels and short stories exploring the
interaction between humans and robots. The author states that Asimov’s works demonstrate
his belief in self-determination, human liberty, and freedom. He valued these ideals more as
he was from a country where these were denied.
Genre: Literature
Number of words: 405
Type of question: Indirect/Inferential
Refer to the lines of paragraph 4 where the author says, ‘In his three-book “Galactic Empire”
series, his characters argue strongly for self-determination and against allowing a bigger entity
to dictate how people in a smaller land should live their lives. Given Soviet domination of
Eastern Europe after World War II, stories employing those themes would have earned
Asimov, at minimum, a stay in Siberia.’
So, the author claims that Asimov’s characters speak up for self-determination, which would
not have been allowed in the Soviet Union.
Therefore, we can infer that the purpose is to support his claim that Asimov supported these
values.
This is best expressed in option C. Hence, option C is the correct answer.
4. B
Sol. Passage summary
The passage is about Asimov and his works. The passage starts with a description of
Asimov’s Three Laws of Robotics. He wrote several novels and short stories exploring the
www.byjusexamprep.com

interaction between humans and robots. The author states that Asimov’s works demonstrate
his belief in self-determination, human liberty, and freedom. He valued these ideals more as
he was from a country where these were denied.
Genre: Literature
Number of words: 405
Type of question: Direct question
Refer to the lines of paragraph 2, where the author describes psychohistory as ‘an attempt to
predict the future using a combination of sociology and mathematics’.
This is the description given in option B.
Hence, option B is the correct answer.
5. A
Sol. Passage summary
The passage is about Asimov and his works. The passage starts with a description of
Asimov’s Three Laws of Robotics. He wrote several novels and short stories exploring the
interaction between humans and robots. The author states that Asimov’s works demonstrate
his belief in self-determination, human liberty, and freedom. He valued these ideals more as
he was from a country where these were denied.
Genre: Literature
Number of words: 405
Type of question: Indirect/ Inferential question
Refer to the following laws mentioned in the first paragraph:
1) A robot may not injure a human being or, through inaction, allow a human being to come to
harm.
2) A robot must obey the orders given to it by human beings except where such orders would
conflict with the First Law.
3) A robot must protect its own existence as long as such protection does not conflict with the
First or Second Law.’
The first law is about not harming a human being, the second is about obeying humans, and
the third is about a robot’s right to self-protection, as long as it does not go against the first two
www.byjusexamprep.com

laws.
Hence, option A is the correct answer.
6. C
Sol. Loquacious means talkative.
7. B
Sol. Ineluctable means unavoidable or inevitable.
8. A
Sol. Obsequious means eager to praise or obey someone. ‘Servile’ is the correct synonym.
Hence, option A is the correct answer.
9. C
Sol. A charlatan is a hoax or an imposter.
Quack means a person who dishonestly claims to have special knowledge and skill in some
field, typically medicine. It is the correct synonym. Hence, option C is the correct answer.
10. D
Sol. Ostentation means showiness or flamboyance. Hence, option D is the correct answer.
11. B
Sol. Venerate means having great respect for someone. Hence, option B is the correct answer.
12. D
Sol. Emaciated means very thin and weak. Scrawny is the correct synonym. Hence, option D is the
correct answer
13. B
Sol. Impeccable means flawless or perfect. Hence, option B is the correct answer.
14. A
Sol. Facetious means flippant, not serious about something. Frivolous is the correct synonym.
Hence, option A is the correct answer.
15. C
Sol. To quash means to cancel or repeal. Hence, option C is the correct answer.
16. D
Sol. Extraneous means irrelevant. External is a synonym. Hence, option D is the correct answer.
www.byjusexamprep.com

17. D
Sol. Options A, B, and C have errors in comparison.
Option A should be ‘Sachin is more talented and dedicated than Vinod’.
Option B should be ‘I prefer tea to coffee’.
Option C should be ’Students find English easier than Mathematics’.
Hence, option D is the correct answer.
18. B
Sol. The sentence indicates that the rain happened before the tree feel. Hence, we have to use the
past perfect tense for the earlier action, that is, the rain. Hence, option B is the correct answer.
19. A
Sol. Option A is the correct answer as the correct phrase is ‘send someone to Coventry’, which
means ‘refuse to associate with or speak to someone’. Coventry is an industrial city in the
west Midlands of England.
20. C
Sol. The correct answer is option C, as the phrase ‘a spanner in the works’ means ‘a person or
thing that prevents the successful implementation of a plan’.
21. D
Sol. Look after means to take care of. Option A is correct.
Look down on means to be contemptuous of. Option B is correct.
Look for means to look for something or expect something. Option C is correct.
Look forward to means you want it to happen because you think you will enjoy it. Option D is
the incorrect usage of the phrasal verb. In this case, the correct phrase is look on.
Hence, the correct answer is option D.
22. B
Sol. Option B is the correct answer because the correct phrase is ‘greasy spoon’, which refers to ‘a
cheap, run-down cafe or restaurant serving fried foods’.
23. C
Sol. Option C is the correct answer because the phrase ‘grist to the mill’ means ‘useful experience,
material, or knowledge’.
www.byjusexamprep.com

24. C
Sol. Pun has not been used in this line taken from the poem ‘Ode to the West Wind’ by PB Shelley.
- Alliteration has been used in ‘wild West Wind’
- Metaphor: Comparison of the wild West Wind to the breath of Autumn's being
- Personification: Addressing the wind as a person
25. D
Sol. Hyperbole is the correct answer because eating a horse is an exaggeration.
26. D
Sol. Army: A group of frogs
Tower: A group of giraffes
Band: A group of gorillas
Hence, option D is the odd one out.
27. C
Sol. Except for option C, the rest of the answer options are related to the game of lawn tennis.
Hence, option C is the correct answer.
28. C
Sol. Except for option C, the rest of the answer options are modes of transportation. Ballistic refers
to the projectiles and their flight. Hence, option C is the correct answer.
29. D
Sol. Except for option D, the rest of the words mean the dead body of a human or an animal.
Hence, option D is the correct answer.
30. B
Sol. Except for option B, the rest of the words are synonyms, which refers to the examination or
observation of one's own mental and emotional processes. Hence, option B is the correct
answer.
31. B
Sol. The phrase ‘no sooner’ is used to emphasise the short time in which something happens.
Here, the short time refers to the fact that it has only been a while since the COVID-19
www.byjusexamprep.com

pandemic slowed down, and now we have monkeypox to deal with. Hence, the phrase ‘no
sooner’ is appropriate here, and it is the correct answer.
32. B
Sol. Here, the main confusion would lie between options A and B. But option B is the correct
answer in the given context. First of all, monkeypox has not become a pandemic as of yet. So,
it does not make any sense to use the word here. And secondly, endemic refers to a disease
or condition regularly found among particular people or in a certain area. And since the author
is discussing the regions where monkeypox has been found just before the blank, it makes
sense to use the word here. Hence, option B is the correct answer.
33. C
Sol. The preposition ‘with’ is used when one is accompanying something or is in the company of
something. Here, the word is used to indicate that several countries are under the attack of
monkeypox. Hence, option C is the correct answer.
34. D
Sol. Here, we need to understand the underlying meaning of the context. Since we are dealing with
effective management of the community, it would only be possible if people suffering from
monkeypox come forward and get themselves treated. This would set an example for others to
come forward and help the community to reduce the transmission of monkeypox. Now, only if
people are unafraid they would come forward. Hence, option D is the correct answer.
35. C
Sol. The word ‘rhetoric’ refers to the art of effective or persuasive speaking or writing, especially
the exploitation of figures of speech and other compositional techniques. Now, this word is the
most appropriate here because the context wants to convey that if we indulge in racist and
homophobic talks, then the condition regarding monkeypox could worsen even more. Hence,
option C is the correct answer.
36. B
Sol. Here, we will need to fill the blank with an indefinite article because we do not know about
‘attachment therapy’ and that it is being mentioned in the passage for the first time. Also, we
use ‘an’ with a vowel sound. Hence, option B is the correct answer.
www.byjusexamprep.com

37. B
Sol. The answer to this question lies in the understanding of the context. And for that, we need to
revisit the sentence given below:
‘While there, Candace underwent an intervention that’s supposed to ___(37)___ the birthing
process. Therapists wrapped her in a flannel sheet and covered her with pillows to simulate a
womb or birth canal.’
Now, since therapists are trying to simulate a womb or a birth canal, it means they are looking
to simulate the birthing process. ‘Simulate’ means to imitate the appearance or character, and
here, the therapists are doing the same with a birth canal. Hence, option B is correct.
38. A
Sol. The phrase ‘fight out’ refers to fighting, arguing, or competing until an argument or competition
has been settled. Since four people tried to stop her from coming out of the simulated birth
canal, she needed to fight her way out of the situation. Hence, option A is the correct answer.
39. A
Sol. The preposition ‘of’ is used to indicate an association between two entities. Here, Candace’s
death is due to suffocation. The most appropriate preposition here would be ‘of’. Hence, option
A is the correct answer.
40. B
Sol. Psychology is the scientific study of the human mind and its functions, especially those
affecting behaviour in a given context.
Here, Candace’s behaviour was tested under a given context, and hence, option B is the
correct answer.
41. C
Sol. Part II is wrong here. The sentence is stating about the ‘economic’ situation of our country
whereas ‘economical’ means cheap or inexpensive. Hence, option C is the correct answer.
42. D
Sol. Part II is incorrect, as based on the context of the given sentence, it states that many
unnecessary policies are present. ‘Founded’ means something which is set up or established.
It should be ‘found’, which means ‘finding (something)’.
www.byjusexamprep.com

Part III is also incorrect. While going through the context of the sentence, the policymakers
have ignored the unnecessary policies in the past, and they are still doing it. As it is mentioned
in the succeeding part that no likely change is expected in their approach (the action began in
the past, and it is still going on), the correct tense here will be present perfect continuous
tense ‘have been ignoring’.
Hence, option D is the correct answer.
43. C
Sol. Part II is incorrect here. ’Stood along with’ is incorrect in the context of the given sentence. It
implies that Urmila adhered to her thoughts and ideologies. ‘Stand by’ means adhere to or
abide by something promised, stated, or decided and is the appropriate phrase in this context.
Hence, option C is the correct answer.
44. A
Sol. ‘They’ is used in the nominative case, while in the given sentence’s context, the objective case
should be used. ‘Them’ should be used instead of ‘they’. Hence, option A is the correct
answer.
45. B
Sol. The error is in the second part of the sentence.
‘With’ should be used after ‘do away’ because ‘do away with’ means cancelling. In the context
of the given sentence, the government has cancelled the monthly publication. Hence, option B
is the correct answer.
46. D
Sol. Here, only Part I is wrong because ‘in accordance to’ is used when we have to refer to any
person, whereas ‘in accordance with’ is used to refer to the current situation, which here is
‘availability of the limited seats’.
47. D
Sol. Passage Summary: The passage in hand talks about the Jaipur Literary Festivals and
excitement and controversies surrounding it.
Genre: Literature
Number of words: 445
www.byjusexamprep.com

Type of question: Main idea question


The passage informs about the Jaipur Literature Festival, its controversies, and the changes
observed in the festival over the years.
The purpose of the passage is more towards highlighting controversies and the issues most
audiences and authors face today at the festival.
Option D is the most appropriate answer.
The author just informs us about the scenes behind the festival. His purpose is not to highlight
only the negatives or dissuade anyone from visiting the festival.
Option C is incorrect. This option hints at what all kinds of preparation or work may go behind
organising the festival, which is not the case in the passage.
48. C
Sol. Passage Summary: The passage in hand talks about the Jaipur Literary Festivals and
excitement and controversies surrounding it.
Genre: Literature
Number of words: 445
Type of question: Specific idea question
Option A is true. Refer to this line: ’... accused William Dalrymple, a (British) writer who co-
directs the festival, …’
Option B is true and is explicitly mentioned in the first paragraph.
Option C is incorrect. The passage mentions that Hartosh Singh Bal is an Indian editor of a
local magazine. It is not mentioned that he is a writer. Thus, option C is the correct answer.
Option D is true. Refer to this line: ’From January 21st to the 25th a couple of hundred
authors, tens of thousands of book-lovers, and a few Nobel laureates cram the lawns of the
Diggi Palace in the Pink City.’
49. C
Sol. Passage Summary: The passage in hand talks about the Jaipur Literary Festivals and
excitement and controversies surrounding it.
Genre: Literature
Number of words: 445
www.byjusexamprep.com

Type of question: Specific idea question


In the last paragraph, the author of the passage explains that authors may bemoan the
increasing size of the festival.
The reason is not explicitly mentioned in the passage, but through the commentary of Junot
Diaz, it is evident that more audiences may not be good for the writers and their work.
More audiences may induce a capitalistic attitude among the authors and that may influence
the quality of work they do. Their intention would be more towards gaining approval from a
larger section of people.
Further, the author subtly hints at the end of the passage that despite the desire of writers to
have more conversations with the audience, the sheer number of the audience does not seem
to make it happen. The conversation ultimately closes down to gaining approval from the
audience (hinted by the author of the passage at the end by the claps/applause of the
audience).
Thus, both options A and B may be the reason for the lamentation of the authors.
Option C is the correct answer.
50. C
Sol. Passage Summary: The passage in hand talks about the Jaipur Literary Festivals and
excitement and controversies surrounding it.
Genre: Literature
Number of words: 445
Type of question: Specific idea question
Option A is wrong to infer. The passage does not hint at a comparison between the two events
by the delegates.
Option B also cannot be inferred from the passage. It is extreme and there is no supporting
evidence in the passage.
Option C can be inferred. The attack on the Tamil minorities and activists in Sri Lanka was
thought to be worth condemning by the activists. So, delegates visiting the event in a country
where Tamil minorities were attacked, was not taken positively by some activists.
Thus, option C is the correct answer.
www.byjusexamprep.com

Option D cannot be inferred from the passage. The passage suggests that the dispute
between Bal and Dalrymple attracted several discussions of whether or why Indian writers
crave foreign approval, especially from the Brits. It is wrong to infer that the discussion
concluded that Indian writers have lower self-esteem as compared to foreign writers.
51. D
Sol. • Cellular Operators Association of India (COAI) has appointed Pramod K Mittal (President of
Jio Infocomm), as the new Chairperson of the association for 2022-23.
• He will replace Ajai Puri as the COAI Chairperson, who held the position for the two
consecutive terms.
• COAI members include Reliance Jio, Bharti Airtel and Vodafone Idea (VIL).
• Chief Regulatory and Corporate Affairs Officer of Vodafone Idea (VIL), P Balaji named as the
Vice Chairperson of the association.
52. C
Sol. • Tamil Nadu CM, MK Stalin has launched the Ennum Ezhuthum scheme to bridge the
learning gap that was created due to the COVID-19 pandemic among students aged below
eight.
• Aim: To ensure foundational literacy and numeracy by 2025.
• Under this programme, the education department of state will provide workbooks to students
from Classes 1 to 3.
• He has also announced Rs 1,000 incentive for girl students, who are pursuing higher
education.
53. D
Sol. • The Chhatrapati Shivaji Maharaj International Airport (CSMIA) has become India's first
airport to launch a one-of-its-kind Vertical Axis Wind Turbine (VAWT) & Solar PV hybrid (Solar
Mill).
• Aim: Possible utilization of wind energy at the airport.
• This is a pilot program in collaboration with WindStream Energy Technologies India Pvt Ltd.
• WindStream Energy Technologies India Pvt Ltd ensures 24/7 energy generation, harnessing
maximum energy through wind power systems.
www.byjusexamprep.com

54. C
Sol. • Coursera has released the latest Global Skills Report (GSR) 2022, which revealed, India’s
overall skills proficiency has dipped four places to be ranked at the 68th position globally.
• In Asian countries, India stands at the 19th position.
• West Bengal leads the Indian states in terms of skills proficiency.
• For the second consecutive year, Switzerland had the highest-skilled learners followed by
Denmark, Indonesia, and Belgium.
55. D
Sol. • Justice Vipin Sanghi will be appointed as the Chief Justice of the Uttarakhand HC while Justice Rashmin
M Chhaya of the as the Chief Justice of the Gauhati High Court.

• Telangana Chief Justice, Satish Chandra Sharma will be appointed to the Delhi HC.

• Justice Amjad A Sayed and Justice SS Shinde will be elevated as the Chief Justice of the Himachal
Pradesh HC and Rajasthan HC respectively.

• Justice Ujjal Bhuyan will take over as the Chief Justice of the Telangana HC.
56. D
Sol. • The Agricultural and Processed Food Products Export Development Authority (APEDA) has
organized an eight-day long mango festival in Bahrain to boost the export of Mangoes.
• 34 varieties of mangoes from eastern states of West Bengal, Bihar, Jharkhand, Uttar
Pradesh and Odisha, are being displayed at eight different locations of Bahrain’s Al Jazira
group supermarket.
57. D
Sol. • LinkedIn has invested $5,00,000 (₹3.88 crore) in association with UN Women to create employment
opportunities for women.

• The pilot project will launch in Maharashtra to cultivate the digital, soft and employability skills of 2,000
women.

• After the completion of 15-month of the pilot project, the UN Women and LinkedIn will incorporate
lessons learned and evaluative feedback.
58. D
www.byjusexamprep.com

Sol. • The Central Government has nominated Anand Gopal Mahindra (Chairman of Mahindra
group), Venu Srinivasan (TVS Motor’s Chairman), Pankaj Ramanbhai Patel (Zydus
Lifescience’s Chairman ) and Ravindra Dholakia (IIM Ahmedabad’s ex-faculty) as part-time
non-official Directors on the Central Board of RBI.
• The appointment will be for a period of four years and effective from June 14.
• The four new appointees will be part of the 10 non-official directors of the Governing body of
the RBI appointed by the Government from time to time.
• The official full-time directors of the RBI governing board consist of the Governor and four
deputy governors.
59. C
Sol. • US president, Joe Biden has appointed Arati Prabhakar as the head of the White House
Office of Science and Technology Policy.
• She succeeded Eric Lander.
• She would be the first woman and the first person of color to serve as science adviser to the
President of the United States.
• She also served in top roles during the presidential tenures of Bill Clinton and Barack
Obama.
60. A
Sol. • As per the Periodic Labour Force Survey (PLFS) for 2020-21 (July-June), the unemployment
rate of India has declined to 4.2%in 2020-21 (July-June).
• The survey was released by the Ministry of Statistics and Programme.
• The labour force participation rate has been raised to 41.6%.
• The unemployment rate was 4.8% and the labour force participation rate was 40.1% in 2019-
20.
• The PLFS provides quarterly updates for urban areas and an annual report for urban and
rural areas.
61. B
Sol. • The 1st train operated by a private operator between Coimbatore and Shirdi under the
Indian Railways' 'Bharat Gaurav' scheme was flagged off.
www.byjusexamprep.com

• Indian Railways had launched the operation of the theme-based Bharat Gaurav train in the
month of November 2021.

• Aim: to showcase India’s rich cultural heritage and magnificent historical places to the people
of India and the world.

• South Star Rail is the registered service provider that operates this Bharat Gaurav train.

62. D
Sol. • Former Karnataka High Court judge Bhimanagouda Sanganagouda Patil was sworn in as the
Lokayukta of Karnataka.

• Governor Thaawarchand Gehlot administered the oath of office to Justice Patil.

• The post of the head of the anti-corruption ombudsman in Karnataka was vacant after the
end of the tenure of Justice P. Vishwanath Shetty in January 2022, served in the post for five
years.

• He, was elevated to the post of Lokayukta on June 14.

63. C
Sol. • India has witnessed the sharpest rise among the Asian economies, with a six-position jump
from 43rd to 37th rank on the annual World Competitiveness Index 2022.

• The Index has been compiled by the Institute for Management Development.

• Denmark has moved to the top of the 63-nation list from the third position last year, while
Switzerland slipped from the top ranking to the second position and Singapore regained the
third spot from the fifth.

64. B
Sol. • Karnataka swimmer Aneesh Gowda won the highest number of medals (six Gold).

• Maharashtra swimmer Apeksha Fernandes and gymnast Sanyukta Kale won five gold
medals each in the games.
www.byjusexamprep.com

• Host state Haryana becomes the champion at the 4th Khelo India Youth Games 2021.

• Haryana tops the medal tally with 52 gold, 39 silver, and 46 bronze medals.

• While Maharashtra finished 2nd with 45 gold, 40 silver, and 40 bronze.

65. A
Sol. • The Indian govt has introduced the Agnipath Military recruitment scheme, a 4-year tenure
scheme for the defense troops.

• The scheme will facilitate the induction of more troops for shorter-term tenures.

• The scheme has been planned and is being implemented by the Department of Military Affairs.

• Under the scheme, candidates will join the defense forces and continue for a period of four years
only.

• The scheme also aims to induct younger candidates.


66. D
Sol. • The central govt has said that fortified rice will be distributed in 291 aspirational and high-
burden districts across the country through Public Distribution System in 2022-23

• Govt has planned to provide 175 lakh MT of fortified rice during this phase.

• The focus will be on to cover all Integrated Child Development Services Centre, PM - Poshan
and Mid-Day Meal schools.

• During 2021-22, around 17 lakh MT of fortified rice have been distributed under ICDS and
PM- Poshan.

67. B
Sol. * United Nations General Assembly (UNGA) has adopted an India-sponsored resolution on
multilingualism that mentions the Hindi language for the first time.

* The resolution urges United Nations to continue spreading vital communications and
messages in both official and non-official languages, including Hindi.
www.byjusexamprep.com

68. A
Sol. * Amravati to Akola section is a part of NH 53, which connects major cities like Kolkata,
Raipur, Nagpur, and Surat.

* National Highways Authority of India (NHAI) has created a Guinness World Record for the
longest continuously laid bituminous lane of 75 kilometres in 105 hours and 33 minutes
between Amravati and Akola districts in Maharashtra.

69. C
Sol. * Union Minister Jitendra Singh has released 2nd edition of the National e-Governance
Service Delivery Assessment 2021 (NeSDA 2021).

* It covers services across 7 sectors: Finance, Labour and Employment, Education, Local
Governance and Utility Services, Social Welfare, Environment and Tourism sectors.

70. A
Sol. • Ex-registrar of MSU, N J Ojha has been appointed as ombudsman under Mahatma Gandhi
National Rural Employment Guarantee Scheme (MGNREGA)
• He will have a tenure of 2-year term.
71. C
Sol. • Ramkrishna Mukkavilli is the first Indian, who has been recognized by United Nations Global
Compact (UNGC) as a Global Sustainable Development Goal (SDG) Pioneer for Water
Stewardship
• He is the founder and MD of Maithri Aquatech.
72. A
Sol. • WHO has awarded Resource Centre for Tobacco Control (e-RCTC) of the Department of
Community Medicine and School of Public Health at Postgraduate Institute of Medical
Education and Research (PGIMER) in Chandigarh with Regional Director Special Recognition
Award.
73. D
www.byjusexamprep.com

Sol. • India beat Poland by 6-4 to win the final of the inaugural FIH Hockey 5s championship in
Lausanne in Switzerland.
• India had topped the five-team league standings with three wins and one draw en route final,
ending their campaign with an unbeaten record.
• Earlier, India has outplayed Malaysia by 7-3 before beating Poland 6-2 in the second match
of the day.
• Hockey 5s was first played in the preparation for the 2014 Youth Olympic Games.
74. B
Sol. • Chief Minister of Goa, Pramod Sawant has launched the “Beach Vigil App”.
• Aim to ensure the holistic management of beaches.
• This app shall benefit the institutions working in the beach tourism sector and tourists.
• With the help of this app, the Drishti workers, police, and other stakeholders can raise issues
for safeguarding tourists’ interests.
• The state government also plans to make the coastal state among the top 25 destinations for
start-ups in Asia by 2025.
75. A
Sol. • QS World University Ranking 2023 has been released, in which 41 Indian varsities have been found
place in the rankings.

• Indian Institute of Science (IISc) ranked 155th as the top research university in the world with a perfect
score of 100 in the citation per faculty parameter.

• IIT Bombay placed at the 172nd rank and IIT Delhi has been ranked 174.

• Massachusetts Institute of Technology (MIT) at top position followed by the University of Cambridge,
Stanford University
76. D
Sol. • International Monetary Fund (IMF) has appointed Krishna Srinivasan as Director of the Asia
and Pacific Department (APD) with effect from June 22.
• He will succeed Changyong Rhee whose retirement from the IMF was announced on March
23.
www.byjusexamprep.com

• He has joined IMF in 1994 in the Economist Program.


77. D
Sol. • Bajram Begaj has been elected as the new president of Albania, with 78 votes in favor, four
against and one abstained at a parliament session.
• Prior to this, he holds the position of Chief of General Staff of the Albanian Armed Forces
(AAF).
• He is the 8th president of Albania and the 3rd from the military ranks.
• He will take oath as the new president on 25th July 2022 to replace the current president, Ilir
Meta.
78. B
Sol. • To reduce the corruption at government offices in the State, Chief Minister YS Jagan Mohan
Reddy has launched an app - ACB 14400.
• This app is developed by the Anti-Corruption Bureau (ACB).
• With the help of this app, people can lodge corruption-related complaints against officials in
the State.
• Earlier, people file a complaint via the toll-free number (14400), but ACB cannot get enough
evidence regarding the complaint.
79. D
Sol. • MTV announced that Jennifer Lopez would be awarded the Generation Award at their forthcoming MTV
Movie & TV Awards.

• MTV Movie & TV Awards recognizes actors who have diversely contributed to both film and TV industry
and are known in every household.

• Vanessa Hudgens will host the Generation Awards.

• Jennifer Lopez will receive the lifetime achievement award.


80. B
Sol. * Vicky Kaushal has won the best actor award for his movie Sardar Udham at the 22nd
International Indian Film Academy Awards in Abu Dhabi, United Arab Emirates.

* Other awards:
www.byjusexamprep.com

* Best Actor (Female): Kriti Sanon (Mimi)

* Best Director: Vishnuvaradhan (Shershaah)

* Best Film: Shershaah

* Best Supporting Actor Female: Sai Tamhankar (Mimi)

* Best Supporting Actor Male: Pankaj Tripathi (Ludo)

* Best Playback Singer Female: Asees Kaur (Raataan Lambiyan, Shershaah)

81. A
Sol. If there is a need to keep a huge liquid base, especially for emerging market economies such
as India, it can definitely be because extreme cases like the COVID-19 pandemic or global
financial crisis could dent forward economic momentum. And since Statement I is a strong
reason behind keeping liquid reserves, it can be a conclusion.
Statement II cannot be a conclusion because we usually keep a buffer to fight off worst-case
scenarios, and here, there is no such thing. Hence, option A is the correct answer.
82. A
Sol. Since the Reserve Bank of India has lifted the ban on Master Card for acquiring new
customers, with immediate effect, the company would surely enter the debit/credit market with
renewed vigour, thereby increasing competition in the market. Hence, Statement I can
definitely be a conclusion.
On the other hand, II is not a conclusion because of the term ‘financial instruments’. A financial
instrument is not a credit/debit card; it is things such as government bonds, cash, and equity.
Hence, option A is the correct answer.
83. C
Sol. Since the supermoon is an astronomical spectacle, there are high chances it would attract
large crowds at places where there is a clear view of the sky because people would want to
witness it. Hence, I can be concluded. Similarly, Statement II can also be concluded because
many astronomy aficionados would line up outside of lunar observatories to witness the thing.
Hence, option C is the correct answer.
www.byjusexamprep.com

84. D
Sol. First of all, since we are dealing with cosmic stuff that has not been discovered yet, we cannot
say anything about it with certainty. Here, both the conclusions are talking about things that
cannot be concluded from the statement because these are quite certain observations that are
not true in the given context. Hence, option D is the correct answer.
85. C
Sol. Since traces of poliovirus have been found in India from time to time, it is clear that the country
has not achieved polio-free status in the actual sense. Hence, II can be concluded. On the
other hand, since the traces of poliovirus have not led to anything serious, there is clearly no
cause for alarmist hysteria. Hence, option C is the correct answer.
86. A
Sol. Since army professionals risk their lives to fight for the honour of this country, they should be
provided life insurance policies from government funds. Their job is highly risky, and a life
insurance policy would ensure their family gets covered if anything happens to them. Hence,
option A is the correct answer.
87. C
Sol. Here, both the arguments are strong. No matter what is going on in your life, you should
always make it a priority to take care of your health because if someone is not healthy,
everything they earn would go to waste. Hence, option C is the correct answer.
88. B
Sol. Comprehensive development or all round development depends on the development of all
sections. Hence, II is strong.
However, the same cannot be said about Statement I because it is asking us a question
instead of strengthening the case of helping out the underprivileged section of society. Hence,
option B is the correct answer.
89. A
Sol. The intention behind the government’s decision to make admissions free in museums is to
preserve and propagate our cultural heritage. Hence, Statement I is a strong argument.
Hence, option A is the correct answer.
www.byjusexamprep.com

Argument II is not strong as the admission is only free for a week and that would not really
make a huge difference
90. C
Sol. Here, both the arguments are strong, because there is a strong need to come up with solid
reforms on data security. Hence, option C is the correct answer.
91. B
Sol. Since it has been mentioned that unhealthy habits can reduce the efficacy of antibiotics, it can
be inferred that our health is dependent on how healthy we eat because the more we eat
unhealthy food, the more likely it is for us to be unaffected by antibiotics in case of bacterial
infections. This idea is best captured in option B, and hence, it is the correct answer.
92. A
Sol. Since some individuals are concerned about using plant-based protein sources in sports
nutrition products, it can be inferred that this is due to the different nutritional needs of
sportspersons as opposed to others. Since the work of a sportsperson involves a high degree
of physical activity, their nutritional needs are bound to be different from the rest. Hence,
option A can be inferred from the given information.
93. C
Sol. First of all, it’s better to eliminate the obvious. For instance, option D is explicitly present in the
given information. Hence, it is a fact, not an inference. Also, option A can be rejected because
there is no comparison going on between the importance of our ecosystem and the
importance of science. Hence, it is rejected as well. Now, option B is rejected because of the
first part of the given sentence. The first part of option B states that now we need to focus
more on the ecosystem, which would mean that previously, there was no such need. This is
not something that can be inferred and hence, option B is rejected as well.
94. A
Sol. Since it has been explicitly mentioned that greenhouse gas emissions should be subjected to
legal controls, it is quite likely that there has been an excess of greenhouse gases in the US.
Hence, option A is the most appropriate inference here.
95. C
www.byjusexamprep.com

Sol. The first line of the sentence clearly indicates polar bears have a close connection with climate
change and have been affected by it Hence, option C is the right answer.
96. A
Sol. Here, only I is a valid assumption because it is the only statement relevant to the statement t.
Since Qatar Airways has recorded high profit in the last one year and is preparing for a record
surge in travellers- it is assuming that there will be a huge increase in the number of
travellersHence, I is an assumption.
On the other hand, II is not an assumption because we can’t say that Qatar Airways is looking
to expand to other territories. Hence, option A is the right answer.
97. C
Sol. The main idea behind making the comparison with Pakistan is to make us aware of the poor
condition of road transportation in India. The suggestion in the paper that road conditions in
India must be improved shows that the assumption is that improving road conditions will
improve speed.
The second sentence that even Pakistan fared better shows that Pakistan was not expected
to do better than India in terms of average speed. So the assumption is that Inid is expected to
do beeter.
Hence, option C is the right answer here.
98. D
Sol. The statement mentions that information about the Rudram missile remains elusive The given
assumption is a restatement of the given statement. Hence it can’t be called an assumption.
On the other hand, II is not an assumption because we can’t say that the testing of all the
missiles has been kept under wraps because the first two have received adequate attention.
Hence, option A is the right answer here.
99. D
Sol. The statement mentions the severe drought like situation in Afghanistan and the resultant food
crisis
I is a restatement of the first line of the passage and hence not an assumption
www.byjusexamprep.com

II - We cannot say that poor water management is the reason for the water crisis since the
passage states that the country is facing severe drought.
Hence neither of the assumptions are implicit
100. A
Sol. I is clearly an assumption because the statement says that people are afaird that technology
will take away their jobs and then many job losses are already happening in some professions
and On the other hand, II is out of the context and hence, is incorrect. Hence, option A is the
right answer.
101. A
Sol. The crux of the paragraph is the idea that obesity can lead to an unhealthy life, something
which has been strengthened by the fact that obesity is the second biggest preventable cause
of cancer. The main point of the passage is that obesity is a cause of concern in England .
Hence, option A is the right answer.
102. B
Sol. Julian Assange’s extradition to the US is a clear indication that anyone who thinks they can
get away from leaking secret information related to the US are far away from being right. The
given idea is best captured in option B and hence, it is the right answer.
103. A
Sol. The passage is primarily written under the context of a strong resistance shown by Ukraine,
coupled with Russia’s military shortcomings. This has resulted in war dragged for over almost
three months. But in the end, the author has mentioned that the picture has darkened again,
which means the situation is more grim than ever. Hence, it can be said that Ukraine’s
resistance is not likely to sustain much longer. Hence, option A is the right answer.
104. B
Sol. The meaning of the difficult words given in the answer options are as follows :
Indiscretion : the trait of being injudicious, lack of good judgement.
Convention : a custom or way of doing things that is widely accepted and followed.
Voyage : a long journey through water
www.byjusexamprep.com

Logic for the question : the written message from a doctor to tell a patient regarding the use of
medicines. Overdose is the taking of more than what was prescribed. So if we follow the
prescription we do not take overdose.
A similar relationship is shown by indiscretion and convention. So if we follow the convention
we will be will take better decisions. But a failure to follow it will lead to a bad decision.
105. B
Sol. The meaning of the difficult words given in the answer options are as follows :
Parquetry : a patterned in lay used to cover floor
Hay : grass mowed and cured for use as a fodder
Statuary :relating to a suitable statues
Effigy : representation of a person in form of a sculpture
Trench : cut or carve deeply into
Logic for the question : Fresco is a mural done with water colours on wet plaster. So a fresco
is done or painted on wall. A similar relationship is shown by option b. parquetry is use to
cover the floor. Rest all the options can be eliminated.
106. B
Sol.
An agreement entered to beat someone is itself illegal. Thus, an agreement to beat C is an illegal act as
per principle 1.
107. B
Sol. No agreement except an agreement to commit an offence shall amount to a criminal conspiracy unless
some act besides the agreement is done by one or more parties to such agreement in pursuance thereof.
In the above case, the agreement to pay off the debt is legal and no further steps to commit robbery has
been taken by either A or B. Thus, this is not criminal conspiracy.
108. A
Sol. The provisions of Indian Penal Code shall apply to any offence committed by any person on any ship or
aircraft registered in India wherever it may be.
109. B
Sol.
The provisions of Indian Penal Code shall apply to any offence committed by any person on any ship or
www.byjusexamprep.com

aircraft registered in India wherever it may be. Thus, where A, who is not an Indian citizen beat B outside
India, though B is an Indian citizen, he cannot be tried in India under the code as per the principle.
110. C
Sol. As per the principle, any person is convicted of an offence for which under this Code the Court has power
to sentence him to rigorous imprisonment, the Court may, by its sentence, order that the offender shall be
kept in solitary confinement for any portion or portions of the imprisonment to which he is sentenced. In
the above case, the court cannot convict the accused for solitary confinement as the offence committed is
punishable for maximum 3 years ‘simple’ imprisonment.

111. C
Sol. Arrest without warrant in a non-cognizable offence because a police officer does not know the correct
procedure of law, is a mistake of law which is not excusable as per the above principle, thus, the police
officer is liable.

112. B
Sol. Arresting B, when C was the correct person to be arrested, considering B as C, is a mistake of fact and A
can be excused under the above principle.
113. C
Sol. Here, A, though guilty of abetting the burning of the house, is not guilty of abetting the theft; for the theft
was a distinct act, and not a probable consequence of the burning.
114. C
Sol. In the above case, the child was acting under the instigation of A, thus, A is liable in the same manner and
to the same extent as if he had instigated the child to put the poison into the food of Y and thus, liable for
abetment to murder.
115. D
Sol. Culpable homicide is not murder if the offender, whilst deprived of the power of self-control by grave and
sudden provocation, causes the death of the person who gave the provocation or causes the death of any
other person by mistake or accident. Thus, where A was beating his wife with a lathi, which accidently falls
on the child, he will not be liable for murder but culpable homicide as he accidently caused the death of
the child.

116. A
Sol. Culpable homicide is not murder if the offender, whilst deprived of the power of self-control by grave and
www.byjusexamprep.com

sudden provocation, causes the death of the person who gave the provocation or causes the death of any
other person by mistake or accident provided the provocation is not given by anything done in obedience
to the law, or by a public servant in the lawful exercise of the powers of such public servant. Thus, where
A is lawfully arrested by police and excited to sudden and violent passion by the arrest kills the officer.
This amounts to murder. as provocation is given by a public servant in the lawful exercise of the powers.

117. D
Sol. In kidnapping from lawful guardianship of a child below 16 or 18 years, consent of child is immaterial as it
is really easy to obtain the consent by enticing the same. Thus, A has committed kidnapping.
118. A
Sol. A person to whom money has been paid, or anything delivered, by mistake or under coercion, must repay
or return it. Thus, where A and B jointly owe Rs.1000 to C, and A pays the whole amount to C. B, not
knowing this fact, pays Rs.1000 again to C. Here C is bound to repay the amount to B received by
mistake.

119. C
Sol. Supplying niece with a number of beautiful dresses cannot be equated with necessaries and thus, the
price of the same cannot be recovered by A.

120. A
Sol. When the person to whom the proposal is made signifies his assent thereto, the proposal is said to be
accepted. A proposal, when accepted, becomes a promise. But, in the above example, the servant did not
know about the offer, and there can be no acceptance before proposal.

121. C
Sol. Coercion is the committing, or threatening to commit, any act forbidden by the Indian Penal Code or the
unlawful detaining, or threatening to detain, any property, to the prejudice of any person whatever, with the
intention of causing any person to enter into an agreement. Threat to divine pleasure cannot be equated
to coercion.

122. A
Sol. All agreements are contracts if they are made by the free consent of parties competent to contract. Thus,
where a contract is made between a minor and A, the minor being incompetent to contract, the contract is
void, irrespective of the fact that A acted bona fide and was deceived by the tall stature of minor.
www.byjusexamprep.com

123. B
Sol. The communication of a revocation is complete as against the person who makes it, when it is put into a
course of transmission to the person to whom it is made, so as to be out of the power of the person who
makes it. Thus, where A revokes his proposal which was made to B by post, the revocation is complete as
against A, when the post is dispatched.

124. C
Sol. A contract is said to be induced by “undue influence” where the relations subsisting between the parties
are such that one of the parties is in a position to dominate the will of the other and uses that position to
obtain an unfair advantage over the other. Thus, where A is indebted to B and the terms of contract seems
to be unfair the contract is voidable at the instance of A.
125. B
Sol. A proposal can be revoked by the death or insanity of the proposer, if the fact of his death or insanity
comes to the knowledge of the acceptor before acceptance. Thus, when A proposes to sell her land to B
for Rs.100000 and goes mad thereafter. B having accepted the proposal before insanity of A, a contract
has been validly concluded.

126. D
Sol. A proposal can be revoked by the death or insanity of the proposer, if the fact of his death or insanity
comes to the knowledge of the acceptor before acceptance. Thus, when A proposes to sell land to B for
Rs.100000 and goes mad, before B could accept the proposal, a contract has not been concluded and
thus, not enforceable.

127. A
Sol. Only one party has promised to pay but no reciprocal consideration has been offered from other side.
Thus, there is no consideration from B’s side to convert the promise into an agreement.

128. C
Sol. The principle of volenti non fit injuria states that the defendant is not liable when the plaintiff consents to
the act which has caused injury to him. Thus, the spectators run the risk of being hit by a ball, and in such
case nobody is liable.

129. A
Sol. The principle of volenti non fit injuria states that the defendant is not liable when the plaintiff consents to
www.byjusexamprep.com

the act which has caused injury to him. The passerby outside the field cannot be said to have consented
to any risk. Thus, the players are liable.

130. B
Sol. Here the doctrine has no application as construction on a busy road mandates to take relevant care of the
passerby. Thus, B cannot be said to have consented running the risk of injury.

131. A
Sol. Trespass means the wrongful disturbance of possession of land or goods of another person. Thus, when
A asked B to leave but, B did not leave. Then, A can sue B for trespass as the lawful entry was turned into
trespass later on.

132. B
Sol. A person who invites another cannot, as and when he wants sue his guest for trespass.

133. C
Sol. Where A regularly cleans the public path adjacent to his house while B with muddy feet walks over the
path just after A mobbed the path. Here, A cannot sue B for trespass as the property is public property
and not his personal property and trespass means the wrongful disturbance of possession of land or
goods of another person.

134. D
Sol. The principle states that damage to a person without legal injury to that person is not actionable in the
court of law. Thus, though damage is caused to A, no legal injury is caused and hence, no compensation
or damages can be claimed by A.
135. A
Sol. The principle states that injury without damage is actionable. Thus, a legal right to vote has been violated
by A, though no damage has been caused to B. Thus, A is liable.
136. C
Sol. Presence of scissors in abdomen of A, proves gross negligence and nothing more to be proved by A.
137. B
Sol. Negligence is a breach of duty or a failure of one party to exercise the standard of care required by law,
resulting in damage to the party to whom the duty was owed. A plaintiff can take civil action against the
www.byjusexamprep.com

respondent, if the respondent’s negligence causes the plaintiff injury or loss of property. Thus, where A
was sick due to food poisoning, there was breach of duty on the part of company.
138. B
Sol. The truck driver was not negligent in driving. A duty to drive with care without risking life is duty of the
every person driving vehicle on road.
139. B
Sol. A tort is a civil wrong. However, murder is a criminal offence.
140. B
Sol. Whoever by words either spoken or intended to be read, or by signs or by visible representations, makes
or publishes any imputation concerning any imputation concerning any person intending to harm, or
knowing or having reason to believe that such imputation will harm, the reputation of such person, is said
to defame that person. Thus, where B told that A stole the shoes, this is not defamation as there was no
intention to harm A’s reputation nor her reputation was actually harmed.
141. B
Sol.
Cricket is played on Tuesday.
Two people play the game between D and the one, who plays Cricket.
A plays the game before Wednesday but does not play Boxing.

Case 1,

Case 2,
www.byjusexamprep.com

Badminton is played before Friday but not on Monday.


C does not play Badminton, which is not played on Wednesday.
Tennis is played after Badminton.

Case 1,

Case 2,

Archery, which is played by E, is played after Boxing, which is not played by B.

Case 2 will be eliminated as A does not play Boxing.

Case 1,
www.byjusexamprep.com

Hence, D plays Tennis on Friday.

142. D
Sol.
9 is placed between each pair of digits of all the numbers where the digits other than 9 are
increasing successively by 1. But the pattern is not followed in 596989. Hence, 596989 is odd
and out.

143. B
Sol. The given code is:

Similarly, we know the following:

The first, second, third and fourth letters move to the third, fourth, seventh and eighth place
and the Fifth, sixth, seventh and eighth letters move to the sixth, fifth, second and first place
respectively.
Therefore, the code for ‘ACCURATE’ is as follows:

144. D
Sol.
Option A: + and –
15 + 2 – 15 ÷ 5 2 = 17 – 6 = 11
Option B: – and ÷
15 ÷ 2 + 15 – 5 2 = 7.5 + 15 – 10 = 22.5 – 10 = 12.5
www.byjusexamprep.com

Option C: × and +
15 – 2 × 15 ÷ 5 2 = 15 – 6 + 2 = 11
Option D: – and ×
15 2 + 15 ÷ 5 – 2 = 30 + 3 – 2 = 31
145. A
Sol. Looking at the options:

Option A: Q * V means Q is the mother of V. V + O means V is the brother of O.

O ^ T means that O is the son of T. Hence, T is the husband of Q.

We need not look any further.


146. C
Sol. The given alphanumeric series is as follows:

Hence, option C is correct.

147. D
Sol. M * W means M is the mother of W. W = H means W is the wife of H. H + S means H is the brother of S. S
^ D means S is the son of D.

So, S is the brother of the son-in-law of M. Hence, option E is the correct answer.
148. B
Sol.

So, the earlier position of Aayush was 9th from the right end. Hence, option B is correct.

149. C
www.byjusexamprep.com

Sol.
Angle traced by hour hand in hours = = 250⁰

Angle traced by minute hand in 20 minutes = = 120⁰

Required angle = 250⁰ - 120⁰ = 130⁰

Hence, option C is correct.

150. A
Sol.
Steps to be followed:
(i) Divide the word into two groups containing equal number of letters in each
(ii) Write the letters of the second group in reverse order at first and then write the letters of the first group
in the same order as they were.

For instance, PUR and PLE are two groups derived from PURPLE. PLE is written as ELP and PUR is
written as PUR. Hence, ELPPUR is derived.

Similarly, AMETHYST would be written as TSYH and AMET, and hence the answer would be TSYHAMET.

You might also like